Please confirm topic selection

Are you sure you want to trigger topic in your Anconeus AI algorithm?

Please confirm action

You are done for today with this topic.

Would you like to start learning session with this topic items scheduled for future?

Review Question - QID 213899

In scope icon N/A A
QID 213899 (Type "213899" in App Search)
A 67-year-old man presents to the emergency department with dyspnea that started 1 week ago and has been gradually worsening. His dyspnea is worsened with exertion, and he endorses intermittent 4/10 chest pain at these times. He has had this chest pain for the past year and states it is unchanged and provoked with walking. He denies fevers, chills, nausea, vomiting, or diarrhea. He has a past medical of diabetes, chronic kidney disease, and peripheral vascular disease. His temperature is 97.5°F (36.4°C), blood pressure is 172/92 mmHg, pulse is 100/min, respirations are 27/min, and oxygen saturation is 90% on room air. An ECG is notable for T wave inversions which are similar to a previous ECG. An anterior-posterior chest radiograph is performed as seen in Figure A. An initial troponin is 0.7 ng/mL (normal < 0.4 ng/mL), and a repeat level is 0.6 ng/mL 3 hours later. His troponin at his last hospital visit was 0.7 ng/mL. An echocardiograph is performed and is notable for an ejection fraction of 38% with normal chamber size. The patient is admitted to the floor for high-risk chest pain. Myocardial stenting is performed the following day, and the patient states he is no longer short of breath. A repeat echocardiograph is notable for an ejection fraction of 65%. The patient is able to walk up and down the stairs without shortness of breath, which is new for him. Which of the following is the most likely diagnosis?
  • A
  • A